Quantcast
  • Register
PhysicsOverflow is a next-generation academic platform for physicists and astronomers, including a community peer review system and a postgraduate-level discussion forum analogous to MathOverflow.

Welcome to PhysicsOverflow! PhysicsOverflow is an open platform for community peer review and graduate-level Physics discussion.

Please help promote PhysicsOverflow ads elsewhere if you like it.

News

PO is now at the Physics Department of Bielefeld University!

New printer friendly PO pages!

Migration to Bielefeld University was successful!

Please vote for this year's PhysicsOverflow ads!

Please do help out in categorising submissions. Submit a paper to PhysicsOverflow!

... see more

Tools for paper authors

Submit paper
Claim Paper Authorship

Tools for SE users

Search User
Reclaim SE Account
Request Account Merger
Nativise imported posts
Claim post (deleted users)
Import SE post

Users whose questions have been imported from Physics Stack Exchange, Theoretical Physics Stack Exchange, or any other Stack Exchange site are kindly requested to reclaim their account and not to register as a new user.

Public \(\beta\) tools

Report a bug with a feature
Request a new functionality
404 page design
Send feedback

Attributions

(propose a free ad)

Site Statistics

205 submissions , 163 unreviewed
5,047 questions , 2,200 unanswered
5,345 answers , 22,709 comments
1,470 users with positive rep
816 active unimported users
More ...

  One Particle State in Interacting QFT (4.88 4.89 in Peskin)

+ 0 like - 0 dislike
1071 views

How to derive equation 4.88 in section 4.6, page 108, of Peskin?

$$\left|k_{1}k_{2}\right\rangle\propto\lim_{T\rightarrow+\infty(1-i\epsilon)}e^{-iHT}\left|k_{1}k_{2}\right\rangle_{0}$$

How to derive equation 4.89?

$$\lim_{T\rightarrow+\infty(1-i\epsilon)} {}_{0}\!\left\langle k_{1}k_{2}\right|e^{-iH(2T)}\left|p_{1}p_{2}\right\rangle_{0}$$$$\propto\lim_{T\rightarrow+\infty(1-i\epsilon)}{}_{0}\!\left\langle k_{1}k_{2}\right|T\left(\exp\left[-i\int_{-T}^{+T}dt\, H_{I}(t)\right]\right)\left|p_{1}p_{2}\right\rangle_{0}$$

asked May 13, 2017 in Theoretical Physics by XIaoyiJing (50 points) [ revision history ]
recategorized May 13, 2017 by Dilaton

The first eqn is an assumption (which can only be proven for specific given Hamiltonians), that when two particles are far apart their mutual interaction should be negligible, hence an eigenstate of the free Hamiltonian; the second is the Dyson series, you should be able to find its derivation in QM textbooks, say Sakurai's modern QM. 

@Jia Yiyang Thanks a lot. Does it mean that the interaction term should be $e^{-\epsilon |t|}H_{int}$?

no, $H_{int}+i\epsilon$ for positive times and $H_{int}-i\epsilon$ for negative times.

@Arnold Neumaier Thanks a lot.

@Arnold Neumaier Could you elaborate? 

It is only ''roughly'' equivalent. $t\to\pm\infty(1-i\epsilon)$ says $t=s(1-i\epsilon)$ with $s\to\pm\infty$. Substitute this into $tH$ and note that in the limit where $\epsilon$ vanishes, $H\epsilon$ and $\epsilon$ behave similarly (if $H$ is nonnegative).

Your answer

Please use answers only to (at least partly) answer questions. To comment, discuss, or ask for clarification, leave a comment instead.
To mask links under text, please type your text, highlight it, and click the "link" button. You can then enter your link URL.
Please consult the FAQ for as to how to format your post.
This is the answer box; if you want to write a comment instead, please use the 'add comment' button.
Live preview (may slow down editor)   Preview
Your name to display (optional):
Privacy: Your email address will only be used for sending these notifications.
Anti-spam verification:
If you are a human please identify the position of the character covered by the symbol $\varnothing$ in the following word:
p$\hbar$ysicsOve$\varnothing$flow
Then drag the red bullet below over the corresponding character of our banner. When you drop it there, the bullet changes to green (on slow internet connections after a few seconds).
Please complete the anti-spam verification




user contributions licensed under cc by-sa 3.0 with attribution required

Your rights
...